LSAT and Law School Admissions Forum

Get expert LSAT preparation and law school admissions advice from PowerScore Test Preparation.

 Administrator
PowerScore Staff
  • PowerScore Staff
  • Posts: 8917
  • Joined: Feb 02, 2011
|
#22837
Complete Question Explanation

Main Point. The correct answer choice is (E)

By stating what "some legislators" refuse to do in the beginning of her argument, the author is almost certainly preparing to make a counterargument. Indeed, her conclusion ("such a position ignores the lesson of experience") reveals an opposing point of view: public funding of new scientific research should not be contingent upon the promise that such research will contribute to the public welfare.

Remember: when dealing with a Main Point question, having a solid grasp of the precise nature and scope of the conclusion will be crucial — in fact, it is often the only thing you need to do in order to answer the question correctly.

Answer choice (A): This answer choice is too strong. The author never promised that the commitment of public funds will necessarily enhance public welfare. While such commitment has enhanced public welfare in the past (and the author hopes it will in the future), you should not assume that an outcome is certain just because it is possible or probable.

Answer choice (B): This answer choice is a Mistaken Reversal of the author's premise: just because some legislators require guarantees that the research will contribute to the public welfare does not mean that this guarantee will be sufficient for the commitment of public funds. Certainty of outcome is a necessary but insufficient condition for financial support. Furthermore, even if this answer choice were logically identical with the first sentence in the stimulus, it would not have been a main point.

Answer choice (C): While it is possible that such discoveries would have occurred sooner, this is by no means a guarantee. Furthermore, the author's main point has more to do with the present and future implications of legislative policy, not with what could have happened in the past.

Answer choice (D): At first glance, this seems like an attractive answer. However, committing public funds to new scientific research is not meant to ensure that research is directed toward contributing to the public welfare. In fact, the author's main premise is that such assurances are not always possible and should not be required for the commitment of public funds.

Answer choice (E): This is the correct answer choice. This is an exact paraphrase of the author's main point. See discussion above.
User avatar
 riabobiia
  • Posts: 8
  • Joined: Nov 09, 2023
|
#103985
Hi Powerscore Staff!

I'm still having difficulty understanding why E is the correct answer and B is not. I diagrammed the contrapositive of the conditional given in the stimulus:

Initial Diagram
Legislators cannot be assured that the research will contribute to the public welfare ----> refuse to commit public funds for new scientific research


Contrapositive
Not refuse to commit public funds for new scientific research ----> Legislators can be assured that the research will contribute to the public welfare


Is the reason B is wrong because it's a Mistaken Reversal of the contrapositive? Is that something that's possible?

I was between B and E, but I just thought that B was a better option because the passage included the example of a scientist applying for the funds to study mold, and how no one was able to predict that it would have been such a large scientific achievement and benefit to public welfare. To me, this read as a suggestion that if legislators knew that the study of mold would have eventually been so influential to public welfare, they wouldn't have opposed funding.

"Such a position ignores the lessons of experience. Many important contributions to the public welfare that resulted from scientific research were never predicted as potential outcomes of that research" was ultimately what pushed me towards B.

I can understand how E is correct, I just can't fully grasp how it's a better answer than B.

Any help is appreciated! Thank you so much!
 Luke Haqq
PowerScore Staff
  • PowerScore Staff
  • Posts: 742
  • Joined: Apr 26, 2012
|
#103996
Hi riabobiia!

As the administrator notes above, answer choice (B) is a Mistaken Reversal of the author's premise (yes, to your question, since the contrapositive and the normal statement say the same thing, you can view it as a Mistaken Reversal while looking at the contrapositive). Answer choice (B) gets the sufficient and necessary conditions mixed around; knowing the outcome of scientific research may be necessary, but nothing in the stimulus states that knowing the outcome is sufficient to guarantee that legislators will commit public funds.

In addition, answer choice (B) doesn't get to the main conclusion of the stimulus: "Such a position ignores the lessons of experience." This is captured by answer choice (E). The author of the stimulus unpacks an example of research (mold research) with an outcome that was not expected but greatly benefited public welfare. This example supports the conclusion that the legislators are ignoring the lessons of experience.

Get the most out of your LSAT Prep Plus subscription.

Analyze and track your performance with our Testing and Analytics Package.